Is this statement about the rank of a linear map true or false?

In summary, the conversation discusses the statement that if T: U -> V is a linear map, then Rank(T) <= (dim(U) + dim(V))/2. The participants provide a proof and a counterexample for this statement. The proof uses the dimension theorem and the fact that ran(T) is a subspace of V. The counterexample is based on the non-invertibility of T, which leads to a nullity greater than 0 and a rank smaller than dim(U). Additionally, the concept of rank as the number of pivot columns in a reduced form is mentioned as another approach to prove the statement. Finally, an example is given to demonstrate that the rank can be less than 1 for a map from R to
  • #1
maximus101
22
0
Is this statement true or false

if false a counterexample is needed

if true then an explanation

If T : U [tex]\rightarrow[/tex] V is a linear map, then Rank(T) [tex]\leq[/tex] (dim(U) + dim(V ))/2
 
Physics news on Phys.org
  • #2
Is this a homework problem? If so, it should be posted in the homework section of the forum.

In any case, this looks true to me. Here's my proof.

Suppose U and V are finite-dimensional vector spaces and [itex]T \colon U \to V[/itex] is linear. Since [itex] \text{ran}(T) \subseteq V [/itex] (where ran(T) is the range of T), then rank(T) [itex] \leq [/itex] dimV. Also, by the dimension theorem, dimU = rank(T) + nullity(T). Putting these two facts together, we have

[tex]
\frac{dimU + dimV}{2} \geq \frac{dimU + rank(T)}{2} = \frac{rank(T) + nullity(T) + rank(T)}{2} = rank(T) + \frac{nullity(T)}{2} \geq rank(T)
[/tex]
 
  • #3
Well strickly speaking, spamiam's equation should be read Right to Left, but otherwise this is absolutely true.
 
  • #4
spamiam said:
Is this a homework problem? If so, it should be posted in the homework section of the forum.

In any case, this looks true to me. Here's my proof.

Suppose U and V are finite-dimensional vector spaces and [itex]T \colon U \to V[/itex] is linear. Since [itex] \text{ran}(T) \subseteq V [/itex] (where ran(T) is the range of T), then rank(T) [itex] \leq [/itex] dimV. Also, by the dimension theorem, dimU = rank(T) + nullity(T). Putting these two facts together, we have

[tex]
\frac{dimU + dimV}{2} \geq \frac{dimU + rank(T)}{2} = \frac{rank(T) + nullity(T) + rank(T)}{2} = rank(T) + \frac{nullity(T)}{2} \geq rank(T)
[/tex]


Hey, thank you I understand now. Could you give me an example where [tex](dimU + dimV)/2[/tex] > [tex]rankT[/tex]
 
  • #5
It is not at all difficult to make up examples.
Suppose U and V have the same dimension but T in not invertible. Then (dim U+ dim V)/2= dim U> rank of T

If T is invertible, then V must have dimension at least equal to the dimension of U. If T is invertible and dim(U)= dim(V), then (dim(U)+ dim(V))/2= dim(U)= rank of T but if dim(V)> dim(U), (dim(U)+ dim(V))/2> dim(U)= rank(T).
 
  • #6
HallsofIvy said:
It is not at all difficult to make up examples.
Suppose U and V have the same dimension but T in not invertible. Then (dim U+ dim V)/2= dim U> rank of T


okay, in this case, why is dim U > Rank T ?

I'm not sure how to use the fact that it is non-singular
 
Last edited:
  • #7
If T is not invertible, then it kernel is not trivial (there exist non-zero v such that T(v)= 0) and its nullity (dimension of the kernel) is greater than 0. rank(T)+ nullity(T)= dim(V) (which equals dim(U) in this example) so that rank(T)= dim(U)- nullity(T)< dim(U)/
 
  • #8
HallsofIvy said:
If T is not invertible, then it kernel is not trivial (there exist non-zero v such that T(v)= 0) and its nullity (dimension of the kernel) is greater than 0. rank(T)+ nullity(T)= dim(V) (which equals dim(U) in this example) so that rank(T)= dim(U)- nullity(T)< dim(U)/

great got it, thanks
 
  • #9
this follows from the meaning of dimension and rank.

i.e. if v1,...vn is a basis for U, then the rank of T is the dimension of the span of Tv1,...,.Tvn.thus rankT ≤ min{dimU, dimV} ≤ (dim(U) + dim(V ))/2.
 
Last edited:
  • #10
another proof uses the notion of rank of a matrix as the number of pivot columns in a reduced form. this is obviously no greater than the number of all columns, and since there is at most one pivot per row, also no greater than the number of rows.

try cooking up a matrix defining a map R-->R of rank less than one.
 

Related to Is this statement about the rank of a linear map true or false?

1. Is the rank of a linear map always equal to the number of linearly independent rows or columns?

Yes, the rank of a linear map is always equal to the number of linearly independent rows or columns. This is known as the rank theorem.

2. Can the rank of a linear map be greater than the number of rows or columns in the matrix?

No, the rank of a linear map can never be greater than the number of rows or columns in the matrix. This is because the maximum number of linearly independent rows or columns is equal to the number of rows or columns in the matrix.

3. Is the rank of a linear map affected by elementary row or column operations?

No, elementary row or column operations do not affect the rank of a linear map. This is because these operations do not change the linearly independent rows or columns in the matrix.

4. Is the rank of a linear map always equal to the number of non-zero eigenvalues?

No, the rank of a linear map is not always equal to the number of non-zero eigenvalues. This is because the number of non-zero eigenvalues can be greater than or equal to the rank, but not always equal to it.

5. Can the rank of a linear map be negative?

No, the rank of a linear map can never be negative. This is because the rank is defined as the number of linearly independent rows or columns, which is always a positive value.

Similar threads

  • Linear and Abstract Algebra
Replies
7
Views
1K
Replies
4
Views
932
  • Linear and Abstract Algebra
Replies
2
Views
892
Replies
10
Views
2K
  • Linear and Abstract Algebra
Replies
4
Views
1K
  • Linear and Abstract Algebra
Replies
3
Views
1K
  • Linear and Abstract Algebra
Replies
24
Views
3K
Replies
4
Views
1K
  • Linear and Abstract Algebra
Replies
9
Views
4K
  • Linear and Abstract Algebra
Replies
2
Views
1K
Back
Top